हम इस संभावना को कैसे सीमित कर सकते हैं कि एक यादृच्छिक चर अधिकतम है?


21

मान लीजिए कि हमारे पास N स्वतंत्र यादृच्छिक चर हैं X1 , , Xn साथ परिमित साधन μ1μN और variances σ12 , , \ _ sigma_N ^ 2σN2 । मैं इस संभावना पर वितरण-मुक्त सीमा की तलाश कर रहा हूं कि कोई भी XiXN अन्य सभी Xj , j \ neq i से बड़ा है ji

दूसरे शब्दों में, अगर सादगी के लिए हम मान Xi हैं कि X_i का वितरण निरंतर है (जैसे कि P(Xi=Xj)=0 ), तो मैं इस पर सीमाएँ खोज रहा हूँ:

P(Xi=maxjXj).
यदि N=2 , हम पाने के लिए Chebyshev की असमानता का उपयोग कर सकते हैं:
P(X1=maxjXj)=P(X1>X2)σ12+σ22σ12+σ22+(μ1μ2)2.
मैं जनरल एन के लिए कुछ सरल (जरूरी नहीं कि तंग) सीमाएं Nढूंढना चाहता हूं, लेकिन मैं सामान्य एन के लिए परिणाम (एस्थेटिक रूप से) सुखद परिणाम नहीं खोज पाया हूं N

कृपया ध्यान दें कि चर को iid नहीं माना जाता है। संबंधित कार्य के किसी भी सुझाव या संदर्भ का स्वागत है।


अद्यतन: याद है कि धारणा से, μjμi । फिर हम यहां आने के लिए उपरोक्त सीमा का उपयोग कर सकते हैं:

P(Xi=maxjXj)minj>iσi2+σj2σi2+σj2+(μjμi)2σi2+σN2σi2+σN2+(μNμi)2.
इसका तात्पर्य है:
(μNμi)P(Xi=maxjXj)(μNμi)σi2+σN2σi2+σN2+(μNμi)212σi2+σN2.
यह, बदले में, तात्पर्य:
i=1NμiP(Xi=maxjXj)μNN2i=1N1(σi2+σN2).
मैं अब सोच रहा हूं कि क्या इस बाउंड को कुछ ऐसे में सुधार किया जा सकता है जो एन पर रैखिक रूप से निर्भर नहीं करता है N। उदाहरण के लिए, निम्नलिखित होल्ड करता है:
i=1NμiP(Xi=maxjXj)μNi=1Nσi2?
और यदि नहीं, तो क्या एक प्रतिरूप हो सकता है?

3
इस बाध्य अगर आप इंडेक्स का उपयोग करके तंग किया जा सकता है है कि आप छोटे ऊपरी के बजाय बाध्य देता । ध्यान दें कि यह मान माध्य और विचरण दोनों पर निर्भर करता है। jN

5
@MichaelChernick: मुझे विश्वास नहीं है कि यह सही है। उदाहरण के लिए मान लीजिए कि हमारे पास पर तीन समान वितरण हैं । फिर, अगर मैं गलत नहीं हूँ, , जबकि । मुझे नहीं पता कि क्या आपको , लेकिन फिर वही उदाहरण दिखाता है कि यह अभी भी मान्य नहीं है। [0,1]P(X1<maxjXj)=2/3P(X1<X2)=P(X1<X3)=1/2P(Xi>maxjXj)
MLS

2
@ मिचेल: यह अभी भी सच नहीं है, दुर्भाग्य से। घटनाओं के लिए तय हो गई स्वतंत्र नहीं हैं। Aj={Xi>Xj} i
कार्डिनल

2
@कार्डिनल: अन्य चीजों के अलावा, यह बहु-सशस्त्र डाकुओं से संबंधित है। यदि आप पिछले पुरस्कारों के आधार पर हाथ उठाते हैं, तो कितनी बड़ी संभावना है कि आपने सबसे अच्छी बांह को चुना है (जो कि ऊपर के अंकन में ), और क्या हम एक उप लेने के लिए अपेक्षित नुकसान को सीमित कर सकते हैं -सुंदर हाथ? P(XN=maxjXj)
MLS

2
MathOverflow को क्रॉसपोस्ट किया गया: mathoverflow.net/questions/99313
कार्डिनल

जवाबों:


1

आप बहुभिन्नरूपी Chebyshev की असमानता का उपयोग कर सकते हैं।

दो चर केस

एक ही स्थिति के लिए, बनाम , मैं उसी स्थिति में पहुंचता हूं की 4 नवंबर 2016 की टिप्पणीएक्स 2X1X2

1) यदि तो पी ( एक्स 1 > एक्स 2 ) ( σ 2 1μ1<μ2P(X1>X2)(σ12+σ22)/(μ1μ2)2

(और मुझे आश्चर्य है कि आपकी व्युत्पत्ति के बारे में)

समीकरण की व्युत्पत्ति १

  • नए चर का उपयोग करनाX1X2
  • इसे ऐसे बदलना कि इसका मतलब शून्य पर हो
  • निरपेक्ष मूल्य ले रहा है
  • चेबीशेव की असमानता को लागू करना

P(X1>X2)=P(X1X2>0)=P(X1X2(μ1μ2)>(μ1μ2))P(|X1X2(μ1μ2)|>μ2μ1)σ(X1X2(μ1μ2))2(μ2μ1)2=σX12+σX22(μ2μ1)2

बहुभिन्नरूपी मामला

समीकरण (1) में असमानता को प्रत्येक (ध्यान दें कि ये सहसंबद्ध हैं लिए इसे कई रूपांतरित चर लागू करके एक बहुभिन्नरूपी मामले में बदला जा सकता है ।i < n(XnXi)i<n

इस समस्या का समाधान (बहुभिन्नरूपी और सहसंबद्ध) आई। ऑल्किन और जेडब्ल्यू प्रैट द्वारा वर्णित किया गया है। गणितीय सांख्यिकी के इतिहास में 'ए मल्टीवेरिएट टचेबाइक असमानता', खंड 29 पृष्ठ 226-234 http://projecteuclid.org/euclid.aoms/1177706720

नोट प्रमेय 2.3

P(|yi|kiσi for some i)=P(|xi|1 for some i)(u+(ptu)(p1))2p2

जिसमें चर की संख्या, , और ।t = k - 2 i upt=ki2u=ρij/(kikj)

प्रमेय 3.6 एक तंग सीमा प्रदान करता है, लेकिन गणना करना कम आसान है।

संपादित करें

बहुभिन्नरूपी कैंटेली की असमानता का उपयोग करके एक तेज बाउंड पाया जा सकता है । वह असमानता वह प्रकार है जिसे आपने पहले इस्तेमाल किया था और आपको सीमा जो है से तेज ।( σ 2 1 + σ 2 2 ) / ( μ 1 - μ 2 ) 2(σ12+σ22)/(σ12+σ22+(μ1μ2)2)(σ12+σ22)/(μ1μ2)2

मैंने पूरे लेख का अध्ययन करने के लिए समय नहीं लिया है, लेकिन वैसे भी, आप यहां एक समाधान पा सकते हैं:

AW मार्शल एंड आई। ओल्किन 'ए वन-साइडेड इनएबैलिटी ऑफ द चेबिशेव टाइप' एनाल्स ऑफ मैथमेटिकल स्टैटिस्टिक्स वॉल्यूम 31 पीपी। 488-491 https://projecteuclid.org/euclid.ax/1177705913

(बाद में ध्यान दें: यह असमानता समान सहसंबंधों के लिए है, न कि पर्याप्त मदद के लिए। लेकिन वैसे भी आपकी समस्या, सबसे तेज़ बाउंड को खोजने के लिए, अधिक सामान्य, बहुभिन्नरूपी कैंटली असमानता के बराबर है। मुझे आश्चर्य होगा कि समाधान मौजूद नहीं है)


क्या आप बहुभिन्नरूपी चेबीशेव असमानता का स्पष्ट विवरण प्रदान कर सकते हैं?
whuber

1
मैंने संपूर्ण प्रमेय प्रदान करने वाले समाधान को संपादित किया है।
सेक्सटस एम्पिरिकस

-1

मुझे एक प्रमेय मिला है जो आपकी मदद कर सकता है और आपकी आवश्यकताओं के लिए इसे समायोजित करने का प्रयास करेगा। मान लें कि आपके पास:

exp(tE(max1inXi))

फिर जेन्सेन की असमानता के कारण (ऍक्स्प (?) एक उत्तल कार्य है), हम प्राप्त करते हैं:

exp(tE(max1inXi))E(exp(tmax1inXi))=E(max1in exp(tXi))i=1nE(exp(tXi)

अब आपको अपने रैंडम वेरिएबल का जो भी क्षण उत्पन्न करना है, उसे प्लग इन करना होगा (क्योंकि यह mgf की परिभाषा है)। फिर, ऐसा करने के बाद (और संभावित रूप से) अपने पद को सरल करते हुए) आप इस पद को लेते हैं और लॉग लेते हैं और इसे t द्वारा विभाजित करते हैं ताकि आपको शब्द । तब आप कुछ मनमाने मूल्य के साथ टी चुन सकते हैं (सबसे अच्छा है कि यह शब्द छोटा है ताकि बाध्य तंग हो)। एक्स मैं ( मीटर एक एक्स 1 मैं n एक्स मैं )exp(tXiXiE(max1inXi)

फिर, आपके पास अधिकतम r ns के अपेक्षित मूल्य के बारे में एक बयान है। अब संभावना के बारे में एक बयान प्राप्त करने के लिए कि इस आरवी के अधिकतम मूल्य इस अपेक्षित मूल्य से विचलित हो जाते हैं, आप बस मार्कोव की असमानता का उपयोग कर सकते हैं (यह मानते हुए कि आपका आरवी गैर-नकारात्मक है) या एक और अधिक विशिष्ट आरवी, आपके विशेष आरवी पर लागू होता है।

हमारी साइट का प्रयोग करके, आप स्वीकार करते हैं कि आपने हमारी Cookie Policy और निजता नीति को पढ़ और समझा लिया है।
Licensed under cc by-sa 3.0 with attribution required.